Past LSAT Explained/PrepTest 31

Section I Analytical Reasoning edit

Game One edit

Four boys (F, J, M, P) and three girls (N, R, T) will be assigned to a row of five lockers numbered 1-5.
RULES
1. Each locker must be assigned to either one or two children.
2. Each child must be assigned a locker.
3. Each shared locker must assigned to one girl and one boy.
4. J must share a locker.
5. R cannot share a locker.
6. N's locker cannot be adjacent to T's.
7. F must be assigned to locker 3.

WHAT WE CAN INFER 1. There are three lockers assigned to one child only (R and 2 boys, which could be F, M, or P)
2. There are two lockers that are shared (J, N, and T).
3. Because N and T must share lockers, shared lockers cannot be adjacent to each other.

Question 1 edit

E is the correct answer choice. We found this out by looking at the inferences we made from the rules. If at least one child must be assigned to the locker, that would leave two people who would have to share a locker. We know that R must have her own locker and that J must share a locker (Rules 4 and 5). We also know that since shared lockers must be assigned to one girl and one boy, we know that J will share a locker with either N or T. That's one shared locker down, which leave N or T, one of who must share a locker with one of the following boys F, M, or P. We do not know which boy MUST share a locker out of that group. So that leaves J, N and T as the only children that MUST be assigned to a shared locker.

Question 2 edit

We must find what MUST be true when T is assigned to locker 3 and M is assigned to locker 1 alone. Let's draw a diagram.

1. M
2.
3. F/T
4.
5.

According to Rule 6, N cannot be assigned to lockers 2 or 4. We also know from Rule 4 that J must share a locker and if he is sharing a locker then it has to be N because T is sharing a locker with F, who according Rule 7 must assigned to locker 3. That means that J and N must be assigned to locker 5. Answer choice (B) comes the close to stating that.

Question 3 edit

We must find which answer choice which shows which lockers CANNOT be shared, if all of the boys are assigned to consecutively numbered lockers and J is assigned to locker 5. Let's look at what the diagram would look like:

1.
2. M or P
3. F
4. M or P
5. J

According to the Rules and the Diagram, R would have to be assigned to locker 1. Since R doesn't share a locker, any answer choice that doesn't contain locker 1 is wrong. We've now narrowed the answer choices down to (C) and (D). Now let's look at what other locker CANNOT be shared. Since Juan must share a locker with either N or T, and he is assigned to locker 5, that means that locker 4 CANNOT be shared because N and T cannot have adjacent lockers. This leaves only answer choice (D) as the correct answer choice.

Question 4 edit

At first glance this seems like a difficult question, but it's really not. The trick to Logic Games is to take a deep breath, read the question and know exactly what it says. This question ask you to find the maximum number of lockers J can be be assigned to after R has been assigned a locker. Take a look at the following diagram:
1. R
2. J (possibly)
3. F
4. J (possibly)
5. J (possibly)

According to the above diagram, if we only know where R and F (because of the rules), then we can try to find out all of the possible lockers J could be assigned to. If we were to say that R was assigned to locker 2, J would still have a maximum of three lockers to choose from. (C) is the correct answer choice.

Question 5 edit

This question is asking if the three girls were assigned to the first three lockers, then what MUST be true. Let's take a look at the possible diagram.
1. J/N (or J/T)
2. R
3. F/T (or F/N)
4. M or P
5. M or P
We know that R must be assigned to locker 2 in this scenario, because N and T cannot be assigned to lockers adjacent to each other. We also know that J has to be assigned to locker 1 because according to the rules he must share a locker and locker 3 is at capacity because of F, therefore, (A) is the correct answer choice.
(B) is incorrect because the question is asking for what MUST be true not what COULD be true. Looking at the above diagram, it could be true that N is assigned to locker 3, but it doesn't HAVE to be true.
For answer choice (C) see the above explanation.
(D) is incorrect because it COULD be true that J and T share a locker, but it doesn't HAVE to be. J could easily share a locker with N and it would not break any of the rules.
(E) is just flat out wrong. We know that that the first three lockers belong to J, N, R, T, and F only based on the diagram.

Question 6 edit

This question asks if lockers 1 and 2 are assigned to one boy each and are not shared lockers, then who MUST be assigned to locker 4?
1. M or P
2. M or P
3. F
4.
5.
What do we know? We know that lockers 3, 4, or 5 must be shared, which means N and T must be assigned to two of the three lockers. Since N and T cannot be assigned to lockers that are adjacent to each other, someone must be between them and that leaves R because J must share a locker with either N or T. So R must belong to locker to 4. See the completed diagram below:
1. M or P
2. M or P
3. F/N or F/T
4. R
5. J/T or J/N
Answer choice (C) is the correct answer choice.

Game Two edit

A store carries ten types of CDs, both new and used jazz, opera, pop, rap, and soul. The store is having a sale on SOME CDs.
RULES
1. Used pop is on sale, new opera is not
2. If both types of pop are on sale, then all soul is on sale
3. If both types of jazz are on sale, no rap is on sale
4. If neither type of jazz is on sale, then new pop is on sale
5. If either type of rap is on sale, then no soul is on sale

To make it clearer all of the used CDs will be lower case: j, o, p, r, s and the new CDs will be capitalized: J, O, P, R, S. Now let's make some inferences, shall we?
Rule 2: P and p --> S and s. The contrapositive of this statement would be ~S OR ~s --> ~P (Remember that p is always on sale)
Rule 3: J and j --> ~R and ~r. The contrapositive of this statement would be R or r --> ~J or ~J
Rule 4: ~J and ~j --> P. The contrapositive of this statement would be ~P --> J or j
Rule 5: R or r --> ~S and ~s. The contrapositive of this statement would be S and s --> ~R or ~r
Inference: P and p --> S and s --> ~R or ~r

Question 7 edit

This question asks what could be the complete and accurate list of CDs on sale. These type of questions, you don't necessarily have to make a diagram. You can look at the rules and eliminate any answer choice that breaks the rules.
(A) is incorrect because it violates rule 3. If both types of jazz are sale then no rap can be on sale. As the answer choices, both types of jazz are on sale as well as new rap.
(B) violates rule 5.
(C) violates rule 4.
(D) violates rule 2.
(E) is the correct answer choice by process of elimination. However, if you really want to know why this is the correct answer choice... The contrapositive of rule 4 states that if new Pop is on sale then either new jazz or used jazz is on sale. This answer choice meets that criteria, while also being in compliance with the rules for this game.

Question 8 edit

This asks what must be true if S is not on sale. This is one of the few times that we do not need to draw a diagram because the information is already at hand in the conditional statement diagrams and their contrapositives. Looking at the contrapositive of rule 2. If either soul CD is not on sale, then no new pop is on sale. That HAS to be true because it's part of the rules. (E) is the correct answer choice. Let's take a look at why the other answer choices were wrong.

Question 9 edit

Beware of this question. You have to pay attention. This question is if both types of jazz are on sale then what is the minimum number of NEW CDs that could be on sale. Let's start by assuming that all of the new CDs are sale, and then let's use the rules to eliminate any new CDs that CAN'T be on sale. There are a total of 5 new CDs. We know that New Opera is not on sale, according to rule 1, so that brings us down to four NEW CDs on sale. Looking at the contrapositive of rule 4, we can cross new Pop off the list, so that leaves down to three CDs on sale: new jazz, new rap, and new soul. Looking at the contrapositive of rule 2, if new soul is not on sale, then new pop is not on sale. We already crossed off new pop, and not we can cross off new soul, because by doing so means that we are still in compliance with the rules. That leaves us with two new CDs on sale: jazz and rap. Let's see if we can cross off rap without violating any rules. Take a look at the contrapositive of rule 3. If we kept either type of rap on sale, then it would violate that rule, therefore, rap cannot be on sale. That leaves us down to only one new CD on sale and that would be jazz. Answer choice (A).

Question 10 edit

This question asks what CANNOT be true. There are no restrictions, so we need to go through and eliminate all of the incorrect answer choices.
(A): There is no rule regarding used opera, so it is POSSIBLE that used opera could not be on sale. This by itself would make this answer choice wrong. However, we can look at the statement that neither type of rap is on sale. There is no rule that tells us what happens with neither type of rap is on sale.
(B): If neither type of jazz is on sale, then new Pop must be on sale according to rule 4. Since that would mean that both types of pop are sale (rule 1), then that means all soul is on sale. The first part of this answer choice doesn't violate any of the rules. Looking at the second part of the answer choice, there are no rules governing used opera, we only know that new opera is not on sale. It is very possible that used opera is not on sale as well. Remember we are looking for what CANNOT be true.
(C): Skipping the part about the opera, let's look at the second part of the answer choice that states neither type of soul is on sale. Remember that on the LSAT, either can mean at least on of the two, possible both. So according to the contrapositive of rule 2, if either type of soul is not on sale then new pop is not on sale. We can say that both new and used soul is not on sale, so that means that there is no new pop on sale. Taking a look at the contrapositive of rule 4, if there is no new pop on sale, then either new jazz or used jazz could be on sale. If both new and used jazz are on sale, then no rap could be on sale. This scenario doesn't break the rules.
(D): If neither type of jazz is on sale, then new pop is on sale and if both types of pop are on sale, then all soul is on sale, but this answer choice states that neither type of soul is on sale and that cannot be true. This is the correct answer choice. (E): If neither type of jazz is on sale then new pop is on sale. If both new and used pop are sale then all soul is on sale. If all soul is on sale then neither type of rap could be on sale. This doesn't violate any of the rules. This is not the correct answer choice.

Question 11 edit

This question asks what CANNOT be true if neither type of jazz is on sale. If neither type of jazz is on sale then new pop is on sale. If both new and used pop is onsale then all soul is on sale. If all soul is on sale, then no rrap is on sale. That leaves only answer choice (A) as the only option that CANNOT be true.

Question 12 edit

This question asks what CANNOT be true if new soul is the ONLY new CD on sale. It is important to look at what the question stems states. Getting the details of the question stem wrong could mean that you get the incorrect answer!
If new soul is the ONLY new CD on sale, then both types of rap cannot be on sale according to the contrapositive of rule 5, so that eliminates answer choice (C). If both types of soul are not on sale, then new pop is not on sale, and if new pop is not on sale then either type of jazz is on sale. Since we know that new jazz is not on sale, used jazz HAS to be on sale. Answer choice (A) states that it is NOT on sale and that cannot be true.

Question 13 edit

This question ask what could be true if four out of the five used CDs were the ONLY CDs on sale. Remember to pay attention to the details. We know that used pop is one of the four CDs that would be on sale, so that leaves down to three used CDs. Let’s start with finding which used CDs could not be on sale. We’ll start with used, because Rule 4 is negative. If both types of jazz are not on sale, then new pop is on sale. Well that violates what the question is stating, so used jazz MUST be on sale. That eliminates answer choices (A) and (D). Looking quickly at answer choice (E) we can eliminate that right away because if both used rap and used soul are not on sale, then that violates the conditions set forth in the question. That leaves us down to answer choices (B)and (C). Used opera is a free agent. So let’s skip that for a second. According to rule 5, if used soul is on file, then neither type of rap could be on sale. (C) is the correct answer choice.

Game Three edit

Monday through Friday will only be a total five tours of three divisions, Operations, Production and Sales.
RULES
1. Each division is toured at least once
2. Operations are not toured on Monday
3. Production is not toured on Wednesday
4. Sales if toured on two consecutive days and no other days.
5. If operations is toured on Thursday, then Production is toured on Friday
6. Contrapositive: If Production is NOT toured on Friday, then Operations is not toured on Thursday.
For this particular game, it is best to draw a diagram in order to better visualize the different scenarios required by the questions.

Question 14 edit

(A): Is it possible for the division that is toured on Monday be toured on Tuesday? Well, yes. Sales must be toured on two consecutive days only. That’s the only restriction. So this scenario is entirely possible.
(B): Based on this answer choice, we know that O and S is not the division they are talking about because O cannot be toured on Mondays and S can only be toured on two consecutive days. So that leaves P. Because there are five tours we know that two divisions will be toured twice, one of those will be S, but the other could be either O or P. So it is possible that P could be toured on Monday and Friday.
(C): If the division toured on Tuesday is also toured on Thursday, that would mean S couldn’t be toured on two consecutive days. That would violate the rules and therefore this is the incorrect answer choice.
(D): This means that S has be toured on Monday and Tuesday. Since P cannot be toured on Wednesday, then it means that O is the division that is toured on Wednesday and Friday, which would leave P to be toured on Thursday. Does this break any rules? No. Why? Because according to the contrapositive of rule 5, if P is not toured on Friday then O is not toured on Thursday and this scenario fits that bill.
(E): According to rule 5 it cannot be rule O that is toured on Thursday and Friday, which leaves P and S. Remember P can toured on Friday, without O being toured on Thursday. Remember your conditional reasoning! Let’s say that it is S that is toured on both days. That would mean P could be toured on either Monday or Tuesday or O could be toured on either Tuesday or Wednesday. Both of those scenarios could be true. So this is the incorrect answer choice.

Question 15 edit

This question asks what could be true if Sales and one other division was toured on two consecutive days. You could diagram this easily, but for now, let’s just reason our way through it.
(A): If O is toured on Thursday that means that P is toured on Friday. This answer choice states that P is toured on Monday. We concluded from the rules is that either O or P must be toured twice. For this answer choice, P is toured on Monday and Friday, S would be toured on Tuesday and Wednesday. But the question asks what could happen if besides S, that O or P were toured twice consecutively. This answer choice is proven to be wrong because P is not toured on consecutive days.
(B): This is the correct answer choice. Why? If P is toured on Tuesday and S on Wednesday that means that S is also toured on Thursday. Since O cannot be toured on Monday, due to the rules, it has to be toured on Friday, which means P has to be toured on Monday. So on Monday and Tuesday P is toured and Wednesday and Thursday S is toured and O is toured on Friday.
(C): If O is toured on Tuesday and P is toured on Friday, that means S has to be toured on Wednesday and Thursday. That leaves Monday open for a tour. It can be S because it only has the two consecutive tours and it can't be O because O is not toured on Monday, so that leaves P. The question is looking for the answer choice that shows that either O or P can be toured consecutively along with S, this is not the case.
(D): If S is toured on Monday and O is toured on Friday, that means S is also toured on Tuesday. That leaves P to be toured on Thursday, which leaves O to be toured on Wednesday and Friday. Again this violates what the question is asking.
(E): If S is toured on Wednesday and P is toured on Friday, that means that O cannot give any consecutive tours. It looks like P can do consecutive tours on Thursday and Friday. However, if that happens then O must be toured on Tuesday, which means S cannot be toured on consecutive days. That breaks one of the rules.

Question 16 edit

This question asks which day must P be toured, if there is a division that is toured on Tuesday and Friday. There are two possible scenarios. See Below for Scenario 1: ♥ = day toured; ♦ = day NOT toured

DAY O P S
MON
TUE
WED
THU
FRI


In scenario 1, O is not toured at all. If P is toured on Tuesday and Friday, then S must be toured on Wednesday and Thursday. That leaves Monday free and it has to be O. But wait, O can't be toured on Monday, according to the rules. This scenario is invalid.

DAY O P S
MON
TUE
WED
THU
FRI


In scenario 2, if O is toured on Tuesday and Friday, then on Wednesday and Thursday S must be toured. That leaves Monday open and P can be toured on Monday. There is no scenario 3, because S cannot be toured on days that are not consecutive. According to this scenario, P MUST be toured on Monday, which is answer choice (A).

Question 17 edit

This asks what could be true if a division is toured on Monday and it is NOT toured on Tuesday. So let's draw another diagram:

DAY O P S
MON
TUE
WED X X
THU X X X
FRI X X X


The above scenario is the only scenario that would work. O can't be toured on Monday, and if S is toured on Monday, then MUST be toured on Tuesday, which defies what the question is asking. We did not have to fill in the rest of the diagram because there would be so many different possibilities. Because P has to be toured on Monday, we can eliminate answer choices (A), (B), and (C). Looking at answer choice (D), the question states that if a division is toured on Monday, then it CANNOT be toured on Tuesday, so we can eliminate this answer choice. Late leaves answer choice (E) as the correct answer choice.

Question 18 edit

This question asks what must be true if a division is toured on Tuesday and is also toured on Wednesday. So let's take a look at some diagrams, shall we?:

DAY O P S
MON
TUE
WED
THU X X
FRI X X


DAY O P S
MON
TUE
WED
THU
FRI


According to the only two scenarios, P is always toured on Monday. People couldn't be toured on Tuesday and Wednesday, because P cannot be toured on Wednesday. In both scenarios where both O and S are toured on Tuesday and Wednesday, P is toured on Monday that is answer choice (A).

Game Four edit

A crew of up to five members must complete five tasks in at most three days. The tasks have to be completed in the following order: framing, wallboarding, taping, sanding, and priming.
1. G = taping
2. H = sanding and priming
3. I = framing and priming
4. K = framing and sanding
5. L = wallboarding and taping
6. M = sanding
7. O = wallboarding and priming
RULES
1. at least one task is completed a day
2. taping and priming are completed on different days
3. each member does at least one task
4. each member does no more task a day
5. each task is done by exactly one worker
6. each task is completed the day it is started and before the next task begins.
Let's simplify this a bit. taping = I and K; wallboarding = L and O; taping = G and L; sanding = H, K, and M; priming = H, I, and O.

Question 19 edit

This answer choice which could be a complete and accurate list of members of the crew. In order to find the correct answer choice, we need to looking at the order in which the work can be done. Framing is the first task only I and K can complete framing. this eliminates answer choice (E). Wallboarding is the second task and only L and O can complete this task. This eliminates answer choice (A). Taping is the third task and only G and L can complete this task. This eliminates answer choices (C) and (D). That leaves answer choice (B) as the correct answer.

Question 20 edit

This question asks, if the installation takes three days and if the same two crew members work on the first and third days, then which pair of the crew members who could work on those two days. First we need to find out what work could be done on which days. We know that two people will be working on the first and third days, so that means on the first and third days two tasks must be completed, because according to rules 4 and 5 state that each member does no more than one task a day and each task is done exactly by one worker. The tasks also have to be completed in a specific order. So on Monday, framing and wallboarding have to be completed on Monday and on Friday sanding and priming have to be completed. Let's look at the list of crew members who can work on framing and wallboarding. Framing: I, K and Wallboarding: L, O. Now let's look at the list of crew members who complete sanding and priming. Sanding: H, K, M and Priming: P, I, O. Now we look for the common members which would be I, K, O.
(A): H cannot complete framing and/or wallboarding, so this eliminates this answer choice.
(B): I and K could work on both days, however only both can complete framing but not wallboarding. This eliminates this answer choice.
(C): While the tasks framing and wallboarding can be completed by I and L, L doesn't not complete neither sanding nor priming. This eliminates this answer choice.
(D): K and O can complete framing and wallboarding as well as sanding and priming. This is the correct answer choice.
(E): This leaves framing without anyone to complete this task and only priming would the be only task completed as O is the only one that could work on day three.

Question 21 edit

This questions what us to find the answer choice that cannot be a complete and accurate list of the crew members.
(A): This is the correct answer choice because L or O must be included in the list as they are the only two that can complete the wallboarding task. Neither of them are included in this list and there this cannot be the complete and accurate answer choice.
(B), (C), (D), and (E) are the incorrect answer choices. Remember in order for the list to be complete we need to have either L or O included in the list. Or we need to have either I or K.

Question 22 edit

This answer choice asks what could be a list of all of the crew members that can work on day two, if the installation takes three days and sanding is done on the third day. There are two possible scenarios.
Scenario 1
Day One: framing and wallboarding - [I, K] and [L, O]
Day Two: taping - [G, L]
Day Three: sanding and priming - [H, K, M] and [H, I, O]

According to this scenario, G and L are the only two that can work on day two. Let's look at scenario 2 now.
Scenario 2
Day One: framing - [I, K]
Day Two: wallboarding and taping - [G, L] and [L, O]
Day Three: sanding and priming - [H, K, M] and [H, I, O]
According to this scenario, the list of everyone who can work on day two is G, L, and O. Any answer choice that includes G, L, or O or any combination of thereof. This leads us to answer choice (E).

Question 23 edit

This question asks which could be a pair of members of the crew both of whom work on the same days as each other and perform two tasks. We can use the diagram created for the previous question.
(A): is not the correct answer choice because both only complete one task each.
(B): According to the question, they each have two tasks to complete, that a total of four tasks between the two of them. In this case, H and K complete only three of the four.
(C): Between the two they can complete four tasks. In order for them to work on the same day taping and priming would have to be completed on the same day and that violates rule 2.
(D): This is the correct answer choice. Both K and L can complete a total of four tasks. Both K and L can work on the same day.
(E): Between the two of them they could complete only three tasks.

Section II Logical Reasoning edit

Question 1 edit

Identify the Question Stem:Point at Issue
(D) is the correct answer choice.
Why? Point at Issue questions ask you find the statement or topic that the two speakers disagree with. It means that for every answer choice, one of the speakers has to agree with the statement or topic and the other has to disagree.(D) does this. The Moralist would agree with this statement, while the TV talk show host would not agree with this answer choice.
(A) The moralist doesn't mention censorship in his statements at all. He doesn't agree or disagree. The only person affected is the TV talk show host because he would disagree with that statement.
(B) To some extent both the moralist and the TV talk show host both agree with this statement. Remember, the question stem asks you find what the moralist and TV talk show host disagree with, not what they both agree on.
(C) This statement only affects the moralist as he would agree with that statement. The TV talk show host never mentions the influence of peoples conception of what is the norm.
(E) This statement only affects the TV talk show host as he implies that it would be wrong to not let viewers decide what they want to see. The moralist never talks about this in his statements.

Question 2 edit

Identify the Question Type: Assumption
Identify the Conclusion: "...the search for a parasite has so far been wasted effort."
Pre-phrase: Parasites that kill sweet potato whiteflies will not kill silverleaf whiteflies.
(D) is the correction answer choice.
If we logically negate this statement, it would say that some parasites of the sweet-potato whitefly is also a parasite of the silverleaf whitefly. If that were true then the whole argument would collapse. In order for the argument to be true, No parasite of the sweet-potato whitefly is also a parasite of the sliverleaf whitefly.
(A) According to the stimulus only one variety of the whitefly has RECENTLY become a serious crop pest. No where in the stimulus does it say or imply that all varieties are serious crop pests. If we negate this statement (Not all varieties of the sweet-potato whitefly are serious crop pests), does it have any effect on the argument? No. Sometimes you do not need to negate every single answer choice. Narrow down the answer choices to at most two or three and then negate the sentences
(B) This is a very general conditional statement. The stimulus was very specific and only mentioned that there was ONE type of whitefly that was becoming a serious crop pest. Anytime the stimulus mentions specifics and an answer choice is very general, unless it is a Principle Question, the answer choice is very much likely to be wrong.
(C) Does this statement as it is have any effect on the argument? No. This answer choice is beyond the scope of the argument and is therefore the wrong answer choice.
(E) Regardless of whether or not the entomologist found parasites of the sweet-potato whitefly, it has no bearing on the argument. The stimulus specifically talks about the SEARCH for the parasite.

Question 3 edit

Identify the Question Type: Flaw in Reasoning
Conclusion: There is none
Pre-phrase: Female patients are the best people to poll. (C) is the correct answer choice. The question stem asks you to find the flaw in the program's plan, which is to poll doctors and ask whether female physicians are more sensitive than male physicians to the needs of their female patients. The best person to ask this question are the female patients, not the physicians. Always try to pre-phrase answer choices. It makes finding the correct answer choice quicker and easier
(A) is an incorrect answer choice because as our pre-phrase female patients are the best judges of whether female physicians or male physicians are more sensitive to the needs of female patients.
(B) is an attractive answer choice because how would a woman be able to tell the difference between the type physicians if she has never been treated by a female physician? But the best answer choice still would be (C) because female patients are the best people to poll to get an accurate reading.
(D) is an incorrect answer choice because it is way out of scope. No where in the stimulus is medical research mentioned.
(E) is an incorrect answer choice because again it is out of scope. The stimulus discusses whether or not certain types of physicians are more sensitive to female patients' needs. It does not discuss female patients becoming more involved with managing their medical care.

Question 4 edit

Identify the Question Type: Main Point
Conclusion: "The practice of using this therapy to treat the illness cannot be adequately supported by the claim that any therapy for treating the illness is more effective than no therapy at all."
Pre-phrase: It is not enough to just treat the illness
(E) is the correct answer choice. By finding the conclusion of the argument for Main Point questions, all you need to do is find the answer choice that rephrases the argument's conclusion. In this case, answer choice (E) is the only answer choice that rephrases the conclusion. On most Main Point Questions, it is tricky finding the conclusion, because it is almost never the last sentence in the stimulus and there is almost never any conclusion indicators.
(A) is an incorrect answer choice because the conclusion states that practice of using the therapy cannot be supported by the claim that any therapy for treating the illness is more effective than no therapy at all. This answer choice simple just restates what the author is arguing against.
(B) is an incorrect answer choice because the stimulus doesn't mention anything about the effectiveness of the treatment. This is slightly beyond the scope of the argument.
(C) is an incorrect answer choice because this is a restatement of one of the premises. Anytime an answer choice repeats or rephrases a premise in the stimulus, it will ALWAYS be the incorrect answer choice for Main Point Questions.
(D) is an incorrect answer choice. Remember the stimulus conclusion. The doctor only states that the expense and complication of the treatment must be taken into account. He makes no claim about the effectiveness of the therapy.
(E) is the correct answer choice because it restates the conclusion of the stimulus.

Question 5 edit

Identify the Question Stem: Strengthen/Support
Identify the Conclusion: "...so advertising time will be no harder to sell next fall than it was last fall."
Prephrase: There is something that is happens that makes more people watch television advertising.
Method of Reasoning:Author concludes that selling advertising won't any harder than before because advertisers will continue to profit despite the increased costs in advertising time.
(A): If the costs production and distribution products advertised and along with the increased costs in advertising means that advertisers may not be willing to buy advertising time. This is the incorrect answer choice.
(B): This system for rating audience size is not mentioned in the stimulus. Even if the system changed, the answer choice doesn't specify how it will affect advertisers. This does not strengthen the answer choice.
(C): What does the size of the advertising blocks have on the argument? They don't. This goes beyond the scope of the argument.
(D): The stimulus doesn't differentiate between advertisers who sell products and those that sell services. This is beyond the scope of the argument.
(E): This is the correct answer choice. If people are watching more television, then they are watching more advertising, and if they are watching more advertising, that means that the increased advertising costs are defrayed by the increased number of people watching advertising.

Question 6 edit

Identify the Question Stem:Weaken
Identify the Conclsuion: "...most people can diagnose swimmer's ear in themselves without ever having to consult a physician."
Prephrase: Show that people did consult with a physician before diagnosing themselves.
Method of Reasoning: Author bases the conclusion on a study.
(A): We are looking to undermine the conclusion in the stimulus. This answer choice undermines the person or persons who believe that most people lace the expertise for diagnosing swimmer'sear, which actually strengthens the stimulus's conclusion.
(B): Believing that they had other ailments that they did not have, is beyond the scope of the argument. The argument concerns itself with patients' diagnosing swimmer's ear. This is the incorrect answer choice.
(C): This is the correct answer choice. It's the closes to our prephrase. Even if you were unable to prephrase you should know that this is the correct answer choice. Why? If people had swimmer's ear before and was diagnosed by a physician then when they have again, they may be more likely to correctly diagnose themselves with swimmer's ear the second time around. If that is the case, then the patients needed a physician to diagnose them in the first case. This undermines the conclusion.
(D): Does the stimulus mention anything about doctor's who specialize in ear disease? No. Does this have any bearing on the argument? No. Incorrect answer choice.
(E): Is the stimulus concerned with the treatment of swimmer's ear? No. This is beyond the scope of the argument.

Section III edit

Section IV edit